Schroeders Minkowski-Raum-Integral – Bedenken hinsichtlich Dochtrotationen

Im Anhang von Peskin & Schroeders „An Introduction to Quantum Field Theory“ gibt es eine Liste von Integralen im Minkowski-Raum. Von besonderem Interesse ist für mich das Integral (A.44):

ICH ( Δ ) = D D ( 2 π ) D 1 ( 2 Δ ) N = ( 1 ) N ich ( 4 π ) D / 2 Γ ( N D 2 ) Γ ( N ) ( 1 Δ ) N D / 2

Δ hängt nicht davon ab , und Peskin verwendet die Metrik ( + ) . Auch, D ist offensichtlich die Dimension des Raumes, den wir betrachten. Peskin leitet dieses Integral ab, indem Wick sich so in den euklidischen Raum dreht 0 = ich E 0 , Und 2 = E 2 (dies wird in Kapitel 6.3 näher dargestellt).

(Kontext: Ich muss mir das obige Integral ansehen für N = 3 . Angeblich geht das auseinander D = 4 , das ist, was ich sehe)

Ich habe zwei Fragen zu diesem Integral:

  • Ist das obige Integral aufgrund der verwendeten Wick-Rotation genau oder funktioniert es nur manchmal oder als Annäherung? Ich dachte immer, dass es genau sei, aber kürzlich hatte ich eine Diskussion mit jemandem, der sagte, dass dies nicht stimmt und eine Wick-Rotation nicht unbedingt das genaue Ergebnis liefert. Ich bin verwirrt und hätte gerne eine Klarstellung dazu.

  • Zweitens verwendet Schroeder die ( + ) Metrik (auch bekannt als die falsche Metrik für Ostküsten-Fans wie mich). Das bedeutet, dass 2 = ( 0 ) 2 | | | | 2 . Ich frage mich, ob sich das obige Integral in seinem Ergebnis ändert, wenn wir die Metrik verwenden ( + + + ) ? Meistens rechne ich damit, dass irgendwo ein zusätzliches Minuszeichen rumschwirrt, denn jetzt hätten wir es 2 = ( 0 ) 2 + | | 2 | | .

Seien Sie vorsichtig, wenn Sie Gleichungen von P&S verwenden, da die Ausgaben Fehler enthalten, die auf ihrer Website aufgeführt sind. Ich verbrachte Stunden damit, ein Ergebnis in Frage zu stellen, nur um herauszufinden, dass es auf einen Fehler in P&S zurückzuführen war. In diesem Fall ist das Integral vollkommen in Ordnung.
Schätzen Sie wirklich die Heads-up! Ich wusste nicht, dass P&S eine Website mit Errata hat, das muss ich mir mal ansehen

Antworten (1)

  1. Das euklidische Integral

    (E) ICH E ( Δ )   :=   D D E ( 2 π ) D 1 ( Δ + E 2 ) N   = ( A .44 ) 1 ( 4 π ) D / 2 Γ ( N D 2 ) Γ ( N ) Δ D 2 N
    ist integrierbar gdw
    N > D 2 .
    Siehe auch den oberflächlichen Grad der Divergenz .

  2. Für N < D 2 das euklidische Integral ICH E ( Δ ) wird in der dimensionalen Regularisierung als gleich der rechten Seite deklariert . von Gl. (E) über analytische Fortsetzung .

  3. Das Minkowski-Integral in der ( , ± , , ± ) Die Konvention wird über die Wick-Rotation definiert

    (A.43) M 0   =   ich E 0
    durch das euklidische Integral gegeben sein
    (M) ICH M ( Δ )   :=   D D M ( 2 π ) D 1 ( Δ ± M 2 ) N   := ( E )   ich ICH E ( Δ ) ,
    bzw.